隨機演算

隨機增長模型

  • November 28, 2020

在這個問題中,我們考慮一個隨機增長模型。特別是,考慮以下 SDE 系統:

$$ \begin{align} dX_t &= Y_t dt + \sigma_XdZ_{1t}\ dY_t &= -\lambda Y_t dt + \sigma_Y \rho dZ_{1t} + \sigma_Y\sqrt{1-\rho^2}dZ_{2t}\ X_0 &= 0\ Y_0 &= 0 \end{align} $$

在哪裡 $ Z_{1t} $ 和 $ Z_{2t} $ 是獨立的布朗運動。\

計算 $ E_t[(X_{t+T}-X_t)] $ 和 $ E_t[(X_{t+T} - X_t)^2] $ 作為一個函式 $ T $ 和 $ Y_t $ . 為了說明短期和長期風險之間的區別,計算:

$ \lim_{T \to 0}\frac{E_t[(X_{t+T} - X_t)^2]}{T} $

和 $ \lim_{T \to \infty}\frac{E_t[(X_{t+T} - X_t)^2]}{T} $ \

  1. 兩個獨立的布朗運動之和也是一個布朗運動(這裡正式證明
  2. 首先我們解決 $ Y_t $ . 我們注意到 $ d\bigl( Y e^{\lambda t} \bigr) = dY e^{\lambda t} + \lambda Y e^{\lambda t} $ .

$$ \begin{align} dY_t &= -\lambda Y_t dt + \sigma_Y \rho dZ_{1t} + \sigma_Y \sqrt{1 - \rho^2} dZ_{2t} \ &= -\lambda Y_t dt + \sigma_Y dW_t \ dY_t + \lambda Y dt &= \sigma_Y dW_t \ d\bigl( Y_t e^{\lambda t} \bigr) &= e^{\lambda t} \sigma_Y dW_t \ \Bigl[ Y_t e^{\lambda t} \Bigr]^T_0 &= \sigma_Y \int^T_0 e^{\lambda t} dW_t \ Y_T &= \sigma_Y e^{-\lambda T}\int^T_0 e^{\lambda t} dW_t \ \end{align} $$

我定義的地方 $ W_t = \rho Z_{1t} + \sqrt{1 - \rho^2} Z_{2t} $ . 這是高斯分佈,期望為 0,變異數為 $ {\frac {\sigma^2_Y} {2 \lambda}} \bigl( 1 - e^{-2\lambda T} \bigr) $ 來自伊藤等距

現在插入並解決 $ X_t $ :

$$ \begin{align} dX_t &= Y_t dt + \sigma_X dZ_{1t} \ &= \bigl( \sigma_Y e^{-\lambda t}\int^t_0 e^{\lambda s} dW_s \bigr) dt + \sigma_X dZ_{1t} \ \Bigl[ X_t \Bigr]^T_0 &= \sigma_Y \int^T_0 e^{-\lambda t} \bigl( \int^t_0 e^{\lambda s} dW_s \bigr) dt + \sigma_X Z_{1t} \end{align} $$

我們可以解決 $ \int^T_0 e^{-\lambda t} \bigl( \int^t_0 e^{\lambda s} dW_s \bigr) dt $ 使用按部分隨機積分如此處所做的,使用 $ A = \int^t_0 e^{-\lambda s} ds $ 和 $ B = \int^t_0 e^{\lambda s} dW_s $ 給

$$ \begin{align} \Bigl[ A_t \cdot B_t \Bigr]^T_0 &= \int^T_0 e^{-\lambda t} \bigl( \int^t_0 e^{\lambda s} dW_s \bigr) dt + \int^T_0 \bigr( \int^t_0 e^{-\lambda s} ds \bigr) e^{\lambda t} dW_t \ \int^T_0 e^{-\lambda t} \bigl( \int^t_0 e^{\lambda s} dW_s \bigr) dt &= -\int^T_0 \bigr( \int^t_0 e^{-\lambda s} ds \bigr) e^{\lambda t} dW_t + \bigl( \int^T_0 e^{-\lambda s} ds \bigr) \cdot \bigl( \int^T_0 e^{\lambda t} dW_t \bigr) \ &= -{\frac {1} \lambda}\int^T_0 (e^{\lambda t} - 1) dW_t + {\frac {1} \lambda} (1 - e^{-\lambda T}) \int^T_0 e^{\lambda t} dW_t \ &= -{\frac {1} \lambda}\int^T_0 (e^{\lambda t} - 1) - e^{\lambda t}(1 - e^{-\lambda T}) dW_t\ &= {\frac {1} \lambda}\int^T_0 \bigl(1 - e^{-\lambda (T-t)} \bigr) dW_t \end{align} $$

並將其代入上面,我們有 $$ \begin{align} X_T &= {\frac {\sigma_Y} \lambda}\int^T_0 \bigl(1 - e^{-\lambda (T-t)} \bigr) dW_t + \sigma_X Z_{1T} \end{align} $$

這是兩個(相關)高斯的總和,所以它也是需要的高斯

  1. 從上面,我們有

$$ \begin{align} \Bigl[ Y_s e^{\lambda s} \Bigr]^{T+t}_t &= \sigma_Y \int^{T+t}t e^{\lambda s} dW_s \ Y{T+t} &= e^{-\lambda T} Y_t + e^{-\lambda (T+t)} \sigma_Y \int^{T+t}_t e^{\lambda s} dW_s \end{align} $$

調節 $ Y_t $ ,我們現在可以找到 $ X_{T+t} $ 如上

$$ \begin{align} {\mathbb E}\bigl[(X_{T+t} - X_t \bigr)] &= {\mathbb E}\bigl[ \int_t^{T+t} dX_s \bigr] \ &= {\mathbb E}\bigl[\int^{T+t}_t Y_s ds + \int^{T+t}t \sigma_X dZ{1s} \bigr]\ &= {\mathbb E}\bigl[\int^{T+t}_t Y_s ds\bigr]\ &= {\mathbb E}\bigl[\int^{T}0 Y{u+t} du\bigr] \ &= {\mathbb E}\bigl[\int^{T}_0 \Bigl( e^{-\lambda u} Y_t + e^{-\lambda (u+t)} \sigma_Y \int^{u+t}_t e^{\lambda s} dW_s \Bigr) du \bigr] \ &= {\mathbb E}\bigl[\int^{T}_0 e^{-\lambda u} Y_t du \bigr]\ &= {\frac 1 {\lambda}} Y_t \bigl( 1 - e^{-\lambda T} \bigr) \end{align} $$

(我從哪裡改變了變數 $ s $ 至 $ u = s - t $ ) 這是有道理的- $ Y_t $ 是均值回歸,因此我們預計未來值比目前值更接近零

$$ \begin{align} {\mathbb E}\bigl[(X_{T+t} - X_t \bigr)^2] &= {\mathbb E}\bigl[ \bigl( \int_t^{T+t} dX_s \bigr)^2 \bigr] \ &= {\mathbb E}\bigl[\Bigl(\int^{T+t}_t Y_s ds + \int^{T+t}t \sigma_X dZ{s1} \Bigr)^2 \bigr]\ &= {\mathbb E}\bigl[\bigl( \int^{T+t}_t Y_s ds \bigr)^2 + \int^{T+t}_t \sigma_X^2 dt + 2 \int^{T+t}_t Y_s ds \int^{T+t}t \sigma_X dZ{1s} \bigr]\ &= {\frac {Y_t^2} {\lambda^2}} \bigl( 1 - e^{-\lambda T} \bigr)^2 + \sigma_X^2 T + 2 {\mathbb E}\bigl[ \int^{T+t}_t Y_s ds \int^{T+t}t \sigma_X dZ{1s} \bigr] \end{align} $$

為清楚起見,我將最後一個術語分開: $$ \begin{align} {\mathbb E}\bigl[ \int^{T+t}t Y_s ds \int^{T+t}t \sigma_X dZ{1s} \bigr] &= {\mathbb E}\bigl[ \int^T_0 Y{u+t} du \int^T_0 \sigma_X dZ_{u1} \bigr] \ &= {\mathbb E}\bigl[ \int^T_0 \Bigl( e^{-\lambda u} Y_t + e^{-\lambda (u+t)} \sigma_Y \int^{u+t}t e^{\lambda s} dW_s \Bigr) du \int^T_0 \sigma_X dZ{1u} \bigr] \ &= {\mathbb E}\bigl[ \int^T_0 \Bigl( e^{-\lambda (u+t)} \sigma_Y \int^{u+t}t e^{\lambda s} dW_s \Bigr) du \int^T_0 \sigma_X dZ{1u} \bigr] \ &= {\mathbb E}\bigl[ \int^T_0 \Bigl( e^{-\lambda (u+t)} \sigma_Y \int^{u+t}t e^{\lambda s} \rho dZ{1u} \Bigr) du \int^T_0 \sigma_X dZ_{1u} \bigr] \ &= {\frac {\rho \sigma_Y} {\lambda}} {\mathbb E}\bigl[ \int^T_0 \bigl( 1 - e^{-\lambda T} \bigr) dZ_{1u} \int^T_0 \sigma_X dZ_{1u} \bigr] \ &= {\frac {\rho \sigma_X \sigma_Y} {\lambda}} \int^T_0 \bigl( 1 - e^{-\lambda T} \bigr) du\ &= {\frac {T \rho \sigma_X \sigma_Y} {\lambda}}\bigl( 1 - e^{-\lambda T} \bigr) \end{align} $$

並將其重新插入到我們上面的塊中 $$ \begin{align} {\mathbb E}\bigl[(X_{T+t} - X_t \bigr)^2] &= {\frac {Y_t^2} {\lambda^2}} \bigl( 1 - e^{-\lambda T} \bigr)^2 + \sigma_X^2 T + 2 {\frac {T \rho \sigma_X \sigma_Y} {\lambda}} \bigl( 1 - e^{-\lambda T} \bigr) \end{align} $$

考慮這個過程的行為 $ T \to \infty $ ,我們看到 $ ( 1 - e^{-\lambda T}) $ 項變為零,我們剩下的變異數為 $ \sigma_X^2 T $ ,這只是標準幾何布朗運動的變異數。

作為 $ T \to 0 $ , $ ( 1 - e^{-\lambda T}) \to \lambda T $ 取消了所有的 $ \lambda $ s 所以表達式變成 $$ \begin{align} {\frac 1 T} \lim_{T \to 0} {\mathbb E}\bigl[(X_{T+t} - X_t \bigr)^2] &= Y_t^2 T + \sigma_X^2 - 2 \rho \sigma_X \sigma_Y T \end{align} $$

因此正如預期的那樣,短期變異數為 $ t $ 隨著水平的增加 $ Y_t $ ,如果過程更正相關,則減少。

哇什麼問題!

大問題!首先回憶一下

$$ \begin{cases} dY_{t} = -\lambda Y_{t}, dt + \sigma_{Y} \rho , dZ_{t}^{(1)} + \sigma_{Y}\sqrt{1-\rho^{2}}, dZ_{t}^{(2)} \ Y_{0} = 0. \end{cases} $$ 使用 It^o 微積分來證明 $ d(e^{\lambda t}Y_{t}) = \sigma_{Y}e^{\lambda t} dW_{t}, $ 在哪裡 $ W_{t} = \rho , dZ_{t}^{(1)} + \sqrt{1-\rho^{2}}, dZ_{t}^{(2)} $ 是標準的布朗運動(請記住 $ d[W_{t},Z_{t}^{(1)}] =\rho, dt $ ),並推導出 $$ Y_{t} = \sigma_{Y}e^{-\lambda t} \int_{0}^{t} e^{\lambda s}, dW_{s} $$ 也 $$ X_{t} = \int_{0}^{t} Y_{s}, ds + \sigma_{X}Z_{t}^{(1)}. $$ 我們將首先計算 $ \mathbb{E}{t}[Y{s}] $ 和 $ \mathbb{E}{t}[X{t+T}-X_{t}] $ . 自從 $ e^{\lambda t}Y_{t} $ 是鞅, $$ \mathbb{E}{t}[e^{\lambda s}Y{s}] = e^{\lambda t}Y_{t} \implies \mathbb{E}{t}[Y{s}] = Y_{t}e^{\lambda (t-s)}, $$ 所以 $$ \begin{align*} \mathbb{E}{t}[X{t+T}-X_{t}] &= \mathbb{E}{t} \left [\int{t}^{t+T} Y_{s}, ds + \sigma_{X}(Z_{t+T}^{(1)}-Z_{t}^{(1)}) \right ] \ &= \int_{t}^{t+T} \mathbb{E}{t}[Y{s}], ds \ &= \int_{t}^{t+T} e^{\lambda(t-s)}Y_{t}, ds \ &= \frac{1}{\lambda}Y_{t}(1-e^{-\lambda T}). \end{align*} $$ 現在我們使用 It^o 等距來計算 $ \mathbb{E}{t}[Y{s}Y_{r}] $ : $$ \begin{align*} \mathbb{E}{t} \left [e^{\lambda (s+r)}Y{s}Y_{r} \right ] &= \mathbb{E}{t} \left [\left (e^{\lambda t}Y{t} + \sigma_{Y} \int_{t}^{s} e^{\lambda u}, dW_{u} \right )\cdot \left (e^{\lambda t}Y_{t} + \sigma_{Y} \int_{t}^{r} e^{\lambda v}, dW_{v} \right ) \right ] \ &= e^{2\lambda t}Y_{t}^{2} + \sigma_{Y}^{2} \mathbb{E}{t} \left [\left (\int{t}^{\min{s,r}} e^{\lambda u}, dW_{u} \right )^{2} \right ] \ &= e^{2\lambda t}Y_{t}^{2} + \sigma_{Y}^{2} \int_{t}^{\min{s,r}} e^{2\lambda u}, du \ &= e^{2\lambda t} \left (Y_{t}^{2} + \frac{\sigma_{Y}^{2}}{2\lambda}(e^{2\lambda (\min{s,r}-t)}-1) \right ) \end{align*} $$ 所以 $$ \mathbb{E}{t} [Y{s}Y_{r}] = e^{-\lambda (s+r-2t)}Y_{t}^{2} + \frac{\sigma_{Y}^{2}}{2\lambda}(e^{-\lambda|s-r|}-e^{-\lambda (s+r-2t)}) $$ 也 $$ \mathrm{cov}(Y_{s},Y_{r}) = \frac{\sigma_{Y}^{2}}{2\lambda}(e^{-\lambda |s-r|}-e^{-\lambda (s+r-2t)}). $$ 接下來,計算 $ s \leq r $ $$ \begin{align*} \mathbb{E}{t} [e^{\lambda s}Y{s}(Z_{r}^{(1)}-Z_{t}^{(1)})] &= \mathbb{E}{t} \left [e^{\lambda t}Y{t}(Z_{r}^{(1)}-Z_{t}^{(1)}) + \sigma_{Y} \int_{t}^{s} e^{\lambda u}, dW_{u} \cdot \int_{t}^{r} dZ_{v}^{(1)} \right ] \ &= \sigma_{Y} \rho \int_{t}^{s} e^{\lambda u} , du \ &= \frac{\sigma_{Y}\rho}{\lambda}(e^{\lambda s}-e^{\lambda t}) \end{align*} $$ 和 $$ \mathbb{E}{t} [Y{s}(Z_{r}^{(1)}-Z_{t}^{(1)})] = \frac{\sigma_{Y}\rho}{\lambda}(1-e^{-\lambda (s-t)}). $$ 最後, $$ \begin{align*} &\mathbb{E}{t}[(X{t+T}-X_{t})^{2}] \ &\quad = \mathbb{E}{t} \left [\left (\int{t}^{t+T} Y_{s}, ds \right )^{2} -2\sigma_{X}(Z_{t+T}^{(1)}-Z_{t}^{(1)})\left (\int_{t}^{t+T} Y_{s}, ds \right ) + \sigma_{X}^{2}(Z_{t+T}^{(1)}-Z_{t}^{(1)})^{2} \right ] \ &\quad = \int_{t}^{t+T}\int_{t}^{t+T} \mathbb{E}{t}[Y{s}Y_{r}], ds, dr - 2\sigma_{X}\int_{t}^{t+T} \mathbb{E}{t}[Y{s}(Z_{t+T}^{(1)}-Z_{t}^{(1)})], ds + \sigma_{X}^{2}T \ &\quad = \int_{t}^{t+T}\int_{t}^{t+T} e^{-\lambda (s+r-2t)}Y_{t}^{2} + \frac{\sigma_{Y}^{2}}{2\lambda}(e^{-\lambda|s-r|}-e^{-\lambda (s+r-2t)}), ds, dr \ &\qquad - \frac{2\sigma_{X}\sigma_{Y}\rho}{\lambda} \int_{t}^{t+T} (1-e^{-\lambda (s-t)}), ds + \sigma_{X}^{2}T \ &\quad = \frac{1}{\lambda^{2}}Y_{t}^{2}(1-e^{-\lambda T})^{2} + \frac{\sigma_{Y}^{2}}{2\lambda}\cdot \frac{2(\lambda T + e^{-\lambda T}-1)}{\lambda^{2}} - \frac{\sigma_{Y}^{2}}{2\lambda^{3}}(1-e^{-\lambda T})^{2}\ &\qquad - \frac{2\sigma_{X}\sigma_{Y}\rho}{\lambda} \left (T - \frac{1}{\lambda}(1-e^{-\lambda T}) \right ) + \sigma_{X}^{2}T. \end{align*} $$ 最後,我們使用漸近 $ \frac{1}{\kappa}(1-e^{-\kappa T}) \sim T - \frac{\kappa}{2}T^{2} $ 作為 $ T \rightarrow 0 $ 要得到 $$ \begin{align*} & \frac{1}{T} \mathbb{E}{t}[(X{t+T}-X_{t})^{2}] \ &\quad = \frac{1}{T} \left (Y_{t}^{2}T^{2} + \frac{\sigma_{Y}^{2}T^{2}}{2\lambda } - \frac{\sigma_{Y}^{2}T^{2}}{2\lambda } - \frac{2\sigma_{X}\sigma_{Y}\rho}{\lambda} \cdot \frac{\lambda T^{2}}{2} + \sigma_{X}^{2}T \right ) + \mathcal{O}(T^{2}) \ &\quad = Y_{t}^{2}T + \sigma_{X}^{2} - \sigma_{X}\sigma_{Y}\rho T + \mathcal{O}(T^{2}). \end{align*} $$

引用自:https://quant.stackexchange.com/questions/58729